An air track glider of mass m1 = 0.250 kg moving at 0.900 m/s to the right collides with a glider of mass m2 = 0.500 kg at rest. If m1 rebounds and moves to the left with a speed of 0.300 m/s, what is the speed and direction of m2 after the collision? kinetic energy

Answers

Answer 1

Answer:

The speed of m2 is 0.6 m/s and its direction is to the right.

Explanation:

This numerical can be solved easily by applying law of conservation of momentum to it. According to law of conservation of momentum:

Total Momentum Before Collision = Total Momentum After Collision

m₁u₁ + m₂u₂ = m₁v₁ + m₂v₂

where,

m₁ = Mass of 1st air glider = 0.25 kg

m₂ = Mass of 2nd air glider = 0.5 kg

u₁ =  Speed of 1st air glider before collision = 0.9 m/s

u₂ = Speed of 2nd air glider before collision = 0 m/s (at rest)

v₁ =  Speed of 1st air glider after collision = - 0.3 m/s (negative sign due to change in direction of velocity)

v₂ = Speed of 2nd air glider after collision = ?

Therefore,

(0.25 kg)(0.9 m/s) + (0.5 kg)(0 m/s) = (0.25 kg)(-0.3 m/s) + (0.5 kg)v₂

0.225 kg.m/s + 0.075 kg.m/s = (0.5 kg)v₂

v₂ = (0.3 kg.m/s)/(0.5 kg)

v₂ = 0.6 m/s

Positive sign indicates that v₂ is directed towards right


Related Questions

Let’s consider tunneling of an electron outside of a potential well. The formula for the transmission coefficient is T \simeq e^{-2CL}T≃e ​−2CL ​​ , where L is the width of the barrier and C is a term that includes the particle energy and barrier height. If the tunneling coefficient is found to be T = 0.050T=0.050 for a given value of LL, for what new value of L\text{'}L’ is the tunneling coefficient T\text{'} = 0.025T’=0.025 ? (All other parameters remain unchanged.) Express L\text{'}L’ in terms of the original LL.

Answers

Answer:

L' = 1.231L

Explanation:

The transmission coefficient, in a tunneling process in which an electron is involved, can be approximated to the following expression:

[tex]T \approx e^{-2CL}[/tex]

L: width of the barrier

C: constant that includes particle energy and barrier height

You have that the transmission coefficient for a specific value of L is T = 0.050. Furthermore, you have that for a new value of the width of the barrier, let's say, L', the value of the transmission coefficient is T'=0.025.

To find the new value of the L' you can write down both situation for T and T', as in the following:

[tex]0.050=e^{-2CL}\ \ \ \ (1)\\\\0.025=e^{-2CL'}\ \ \ \ (2)[/tex]

Next, by properties of logarithms, you can apply Ln to both equations (1) and (2):

[tex]ln(0.050)=ln(e^{-2CL})=-2CL\ \ \ \ (3)\\\\ln(0.025)=ln(e^{-2CL'})=-2CL'\ \ \ \ (4)[/tex]

Next, you divide the equation (3) into (4), and finally, you solve for L':

[tex]\frac{ln(0.050)}{ln(0.025)}=\frac{-2CL}{-2CL'}=\frac{L}{L'}\\\\0.812=\frac{L}{L'}\\\\L'=\frac{L}{0.812}=1.231L[/tex]

hence, when the trnasmission coeeficient has changes to a values of 0.025, the new width of the barrier L' is 1.231 L

To increase the energy of an electromagnetic wave, which property should you decrease?
Shift,
Frequency
Speed
Wavelength

Answers

the correct answer is wavelength

The increase in the energy of an electromagnetic wave can be achieved only by decreasing the wavelength. Hence, option (d) is correct.

The given problem is based on the fundamentals of electromagnetic wave and the energy stored in an electromagnetic wave.

The electromagnetic wave stores the energy in the form of radiations also known as the electromagnetic radiations. These radiations can take the several forms such as radio waves, microwaves, X-rays and gamma rays.

The mathematical expression for the energy carried out by the  electromagnetic waves is given as,

[tex]E = h \times \nu\\\\E = \dfrac{h \times c}{ \lambda}[/tex]

Here,

h is the Planck's constant.

[tex]\nu[/tex] is the frequency of the electromagnetic wave.

c is the speed of light.

[tex]\lambda[/tex] is the wavelength of wave.

Clearly, the energy of electromagnetic waves is directly proportional to the frequency of wave and inversely proportional to wavelength. So, decreasing the wavelength, we can easily increase the energy of electromagnetic wave.

Thus, we can conclude that the increase in the energy of an electromagnetic wave can be achieved only by decreasing the wavelength. Hence, option (d) is correct.

Learn more about the electromagnetic wave here:

https://brainly.com/question/3101711

A and B Two vectors are in the xy plane. If 4= 3m , |B|= 4m , and |A– B= 2 m find
a) The angle between B and A
b) The unit vector in the direction of (4× B)​

Answers

Answer:

Explanation:

a)

A = 3m , B = 4m .

(A-B)² = A² + B² - 2ABcosθ where θ is angle between A and B.

4 = 9 + 16 - 2. 3.4 cosθ

cosθ = .875

θ = 29° .

b ) Unit vector in the direction of A X B will be k vector because A and B are in X-Y plane and A X B lies perpendicular to both A and B .

Which electromagnetic wave transfers the least amount of energy?

Answers

Answer:

microwave

Explanation:

For the circuit, suppose C=10µF, R1=1000Ω, R2=3000Ω, R3=4000Ω and ls=1mA. The switch closes at t=0s.1) What is the value of Vc (in volts) just prior to the switch closing? Assume that the switch had been open for a long time. 2) For the circuit above, what is the value of Vc after the switch has been closed for a long time?
3) What is the time constant of the circuit (in seconds)? Enter the answer below without units.
4) What is the value of Vc at t = 2msec (in volts).

Answers

Answer:

1.) Vc = 1V

2.) Vc = 2.7V

3.) Time constant = 0.03

4.) V = 2.53V

Explanation:

1.) The value of Vc (in volts) just prior to the switch closing

The starting current = 1mA

With resistance R1 = 1000 ohms

By using ohms law

V = IR

Vc = 1 × 10^-3 × 1000

Vc = 1 volt.

2.) The value of Vc after the switch has been closed for a long time.

R2 and R3 are in parallel to each other. Both will be in series with R1

The equivalent resistance R will be

R = (R2 × R3)/R2R3 + R1

Where

R1 = 1000Ω,

R2 = 3000Ω,

R3 = 4000Ω

R = (4000×3000)/(4000+3000) + 1000

R = 12000000/7000 + 1000

R = 1714.3 + 1000

R = 2714.3 ohms

By using ohms law again

V = IR

Vc = 1 × 10^-3 × 2714.3

Vc = 2.7 volts

3.) The time constant = CR

Time constant = 10 × 10^-6 × 2714.3

Time constant = 0.027

Time constant = 0.03 approximately

4.) The value of Vc at t = 2msec (in volts). Can be calculated by using the formula

V = Vce^-t/CR

Where

Vc = 2.7v

t = 2msec

CR = 0.03

Substitute all the parameters into the formula

V = 2.7 × e^-( 2×10^-3/0.03)

V = 2.7 × e^-(0.0667)

V = 2.7 × 0.935

V = 2.53 volts

When a star has fused most of its hydrogen and begins to collapse inward, it becomes a

Answers

Red giant. The core collapses inwards increasing the heat and pressure, pushing the star outwards again.
It becomes a black hole or in some cases a white dwarf

Two mirrors are at right angles to one another. A light ray is incident on the first at an angle of 30° with respect to the normal to the surface. What is the angle of reflection from the second surface?

Answers

Answer:

reflected angle - secod mirror = 60°

Explanation:

I attached an image with the solution to this problem below.

In the solution the reflection law, incident angle = reflected angle, is used. Furthermore some trigonometric relation is used.

You can notice in the image that the angle of reflection in the second mirror is 60°

An express train passes through a station. It enters with an initial velocity of 22.0 m/s and decelerates at a rate of 0.150 m/s^{2} as it goes through. The station is 210 m long. (a) How long is the nose of the train in the station? (b) How fast is it going when the nose leaves the station? (c) If the train is 130 m long, when does the end of the train leave the station? (d) What is the velocity of the end of the train as it leaves?

Answers

Answer:

a) [tex]t \approx 9.879\,s[/tex], b) [tex]v = 20.518\,\frac{m}{s}[/tex], c) [tex]t = 16.368\,s[/tex], d) [tex]v = 19.545\,\frac{m}{s}[/tex]

Explanation:

a) Since train is only translating in a straight line and experimenting a constant deceleration throughout the station, whose length is 210 meters. The time required for the nose of the train to reach the end of the station can be found with the help of the following motion formula:

[tex]210\,m = \left(22\,\frac{m}{s}\right) \cdot t + \frac{1}{2}\cdot \left(-0.150\,\frac{m}{s^{2}} \right) \cdot t^{2}[/tex]

The following second-order polynomial needs to be solved:

[tex]-0.075\cdot t^{2} + 22\cdot t - 210 = 0[/tex]

Whose roots are presented herein:

[tex]t_{1}\approx 283.455\,s[/tex] and [tex]t_{2} \approx 9.879\,s[/tex]

Both solutions are physically reasonable, although second roots describes better the braking process of the train.

b) The speed of the nose leaving the station is given by this expression:

[tex]v = 22\,\frac{m}{s} + \left(-0.150\,\frac{m}{s^{2}}\right)\cdot (9.879\,s)[/tex]

[tex]v = 20.518\,\frac{m}{s}[/tex]

c) First, it is required to calculate the time when nose of the train reaches a distance of 130 meters.

[tex]130\,m = \left(22\,\frac{m}{s}\right) \cdot t + \frac{1}{2}\cdot \left(-0.150\,\frac{m}{s^{2}} \right) \cdot t^{2}[/tex]

[tex]-0.075\cdot t^{2} + 22\cdot t - 130 = 0[/tex]

Roots of the second-order polynomial are:

[tex]t_{1} \approx 287.300\,s[/tex] and [tex]t_{2} \approx 6.033\,s[/tex]

Both solutions are physically reasonable, although second roots describes better the braking process of the train. Now, the speed experimented by the train at this instant is:

[tex]v = 22\,\frac{m}{s} + \left(-0.150\,\frac{m}{s^{2}}\right)\cdot (6.033\,s)[/tex]

[tex]v = 21.095\,\frac{m}{s}[/tex]

The distance traveled by the end of the train throughout station is modelled after the following equation:

[tex]210\,m = \left(21.095\,\frac{m}{s}\right) \cdot t + \frac{1}{2}\cdot \left(-0.150\,\frac{m}{s^{2}} \right) \cdot t^{2}[/tex]

[tex]-0.075\cdot t^{2} + 21.095\cdot t - 210 = 0[/tex]

Roots of the second-order polynomial are:

[tex]t_{1} \approx 270.932\,s[/tex] and [tex]t_{2} \approx 10.335\,s[/tex]

Both solutions are physically reasonable, although second roots describes better the braking process of the train. The instant when the end of the train leaves the station is:

[tex]t = 6.033\,s + 10.335\,s[/tex]

[tex]t = 16.368\,s[/tex]

d) The velocity experimented by the end of the train is:

[tex]v = 21.095\,\frac{m}{s} + \left(-0.150\,\frac{m}{s^{2}} \right)\cdot (10.335\,s)[/tex]

[tex]v = 19.545\,\frac{m}{s}[/tex]

The negative sign indicates that the train comes to a stop before entering the station. Therefore, the nose of the train is not in the station at all. The negative sign indicates that the train is moving in the opposite direction of its initial velocity. The end of the train leaves the station after approximately 14.98 seconds. The velocity of the end of the train as it leaves the station is approximately 19.753 m/s.

(a) Using the equation :

v² = u² + 2as

0 = (22.0)² + 2 × (-0.150 ) × s

s = -(22.0 )^2 / (2 × -0.150)

s = 325.3 m

The negative sign indicates that the train comes to a stop before entering the station. Therefore, the nose of the train is not in the station at all.

(b) The velocity of the train when the nose leaves the station is given by:

v = u + at

v = 22.0 + (-0.150 ) × 210

v ≈ 22.0 - 31.5

v ≈ -9.5 m/s

The negative sign indicates that the train is moving in the opposite direction of its initial velocity.

(c)

s = ut + (1/2)at²

210= 22.0 × t + (1/2) × (-0.150) × t²

0.075 t² - 22.0 t + 210 = 0

we find two solutions for t: t ≈ 14.98 s and t ≈ 3.01 s.

The end of the train leaves the station after approximately 14.98 seconds.

(d) The velocity of the end of the train as it leaves the station is given by:

v = u + at

v = 22.0 + (-0.150) × 14.98

v = 22.0 - 2.247

v = 19.753 m/s

So, the velocity of the end of the train as it leaves the station is approximately 19.753 m/s.

To know more about the velocity:

https://brainly.com/question/17127206

#SPJ6

A duck flying horizontally due north at 10.7 m/s passes over East Lansing, where the vertical component of the Earth's magnetic field is 4.09×10-5 T (pointing down, towards the Earth). The duck has a positive charge of 6.47×10-8 C. What is the magnitude of the magnetic force acting on the duck?

Answers

Answer:

2.83×10⁻¹¹ N.

Explanation:

From the question,

Using

F = qvB....................... Equation 1

Where F = magnetic force acting on the duck, q = charge of the duck, v = velocity of the duck, B = magnetic field of the duck.

Given: q = 6.47×10⁻⁸ C, B = 4.09×10⁻⁵ T, v = 10.7 m/s.

Substitute these values into equation 1

F = 6.47×10⁻⁸×4.09×10⁻⁵×10.7

F = 2.83×10⁻¹¹ N.

Hence the magnetic force acting on the duck is  2.83×10⁻¹¹ N.

g A 47.3 kg girl is standing on a 162 kg plank. The plank, originally at rest, is free to slide on a frozen lake, which is a flat, frictionless surface. The girl begins to walk along the plank at a constant velocity of 1.36 m/s relative to the plank. What is her velocity relative to the ice surface

Answers

Answer:

Explanation:

mass of the girl m₁ = 47.3 kg

mass of the plank m₂ = 162 kg

velocity of the girl with respect to surface = v₁

velocity of plank with respect to surface = v₂

v₁+ v₂ = 1.36

v₂ = 1.36 - v₁

applying conservation of momentum law to girl and plank.

m₁v₁ = m₂v₂

47.3 x v₁ = 162 x ( 1.36 - v₁ )

47.3 v₁ = 220.32 - 162v₁

209.3 v₁ = 220.32

v₁ = 1.05 m /s

For saving energy, bicycling and walking are far more efficient means of transportation than is travel by automobile. For example, when riding at 12.5 mi/h, a cyclist uses food energy at a rate of about 360 kcal/h above what he would use if merely sitting still. (In exercise physiology, power is often measured in kcal/h rather than in watts. Here 1 kcal = 1 nutritionist's Calorie = 4186 J). Walking at 3.20 mi/h requires about 220 kcal/h. It is interesting to compare these values with the energy consumption required for travel by car. Gasoline yields about 1.30.
A) Find the fuel economy in equivalent miles per gallon for a person walking.
B) Find the fuel economy in equivalent miles per gallon for a person bicycling.

Answers

Answer:

a. 451.72 mi/ga

b. 1078.33 mi/ga

Explanation:

The computation is shown below:

a. The fuel economy for a person walking is

Given that

Walking at 3.20 mi/h requires about 220 kcal/h so it is equal to

[tex]= 220 \times 4186[/tex]

= 920920 j/hr

Now

[tex]= \frac{mi}{j}[/tex]

[tex]= \frac{3.2}{920920}[/tex]

So,

[tex]= \frac{mi}{ga}[/tex]

[tex]= \frac{3.2}{920920}\times 1.3 \times 100000000[/tex]

= 451.72 mi/ga

b. Now

Bicycling 12.5 mi/h requires about 360 kcal/h  energy so it is equal to

[tex]= 360 \times 4186[/tex]

= 1506960 j/hr

So,

[tex]= \frac{mi}{j}[/tex]

[tex]= \frac{12.5}{1506960}[/tex]

Now

[tex]= \frac{mi}{ga}[/tex]

[tex]= \frac{12.5}{1506960} \times 1.3 \times 100000000[/tex]

= 1078.33 mi/ga

We simply applied the above formula

Electromagnetic waves propagate much differently in conductors than they do in dielectrics or in vacuum. If the resistivity of the conductor is sufficiently low (that is, if it is a sufficiently good conductor), the oscillating electric field of the wave gives rise to an oscillating conduction current that is much larger than the displacement current. In this case, the wave equation for an electric field:________

Answers

Answer:

Del(ρ/ε₀) - (Del)²E = -dμ₀J/dt

Explanation:

From Maxwell's fourth equation

Curl B = μ₀J + μ₀ε₀dE/dt (1) where the second term is the displacement current.

If the oscillation conduction current in the conductor is much larger than the displacement current then, the displacement current goes to zero. So we have

Curl B = μ₀J  (2)(since μ₀ε₀dE/dt = 0)

From maxwell's third equation

Curl E = -dB/dt  (3)  

taking curl of the above from the left

Curl(Curl E) = Curl(-dB/dt)

Curl(Curl E) = (-d(CurlB)/dt)  (4)

Substituting for Curl B into (4), we have

Curl(Curl E) = -dμ₀J/dt

Del(DivE) - (Del)²E = -dμ₀J/dt    (5)

From Maxwell's first equation,

DivE = ρ/ε₀

Substituting this into (5), we have

Del(ρ/ε₀) - (Del)²E = -dμ₀J/dt

When a ball is dropped from a window, how much is the initial velocity in m/s2 ?

Answers

Explanation:

The initial velocity is 0 m/s.

The initial acceleration is -9.8 m/s².

A long solenoid that has 1 080 turns uniformly distributed over a length of 0.420 m produces a magnetic field of magnitude 1.00 10-4 T at its center. What current is required in the windings for that to occur?

Answers

Answer:

Current, I = 0.073 A

Explanation:

It is given that,

Number of turns in a long solenoid is 1080

Length of the solenoid is 0.420 m

It produces a magnetic field of [tex]10^{-4}\ T[/tex] at its center.

We need to find the current is required in the winding for that to occur. The magnetic field at the center of the solenoid is given by :

[tex]B=\mu_0 NI[/tex]

I is current

[tex]I=\dfrac{B}{\mu_o N}\\\\I=\dfrac{10^{-4}}{4\pi \times 10^{-7}\times 1080}\\\\I=0.073\ A[/tex]

There is a known potential difference between two charged plates of 12000 Volts. An object with a charge of 6.5 x 10-6 C charge and a mass of 0.02 kg is placed next to the positive plate. How fast will it be traveling when it gets to the negative plate

Answers

Answer:

1.97 m/s.

Explanation:

From the question,

Using the law of conservation of energy,

The energy stored in the charged plate = Kinetic energy of the mass

1/2(qV) = 1/2mv².......................... Equation 1

Where q = charge, V = voltage, m = mass, v = velocity.

make v the subject of the equation

v = √(qV/m)......................... Equation 2

Given: q = 6.5×10⁻⁶ C, V = 12000 Volts, m = 0.02 kg

Substitute these values into equation 2

v = √(6.5×10⁻⁶×12000 /0.02)

v = √3.9

v = 1.97 m/s.

The concrete slab of a basement is 11 m long, 8 m wide, and 0.20 m thick. During the winter, temperatures are nominally 17 C and 10 C at the top and bottom surfaces, respectively. If the concrete has a thermal conductivity of 1.4 W/m K, what is the rate of heat loss through the slab

Answers

Answer:

Q = - 4312 W = - 4.312 KW

Explanation:

The rate of heat of the concrete slab can be calculated through Fourier's Law of heat conduction. The formula of the Fourier's Law of heat conduction is as follows:

Q = - kA dt/dx

Integrating from one side of the slab to other along the thickness dimension, we get:

Q = - kA(T₂ - T₁)/L

Q = kA(T₁ - T₂)/t

where,

Q = Rate of Heat Loss = ?

k = thermal conductivity = 1.4 W/m.k

A = Surface Area = (11 m)(8 m) = 88 m²

T₁ = Temperature of Bottom Surface = 10°C

T₂ = Temperature of Top Surface = 17° C

t = Thickness of Slab = 0.2 m

Therefore,

Q = (1.4 W/m.k)(88 m²)(10°C - 17°C)/0.2 m

Q = - 4312 W = - 4.312 KW

Here, negative sign shows the loss of heat.

A 58.0 kg skier is moving at 6.00 m/s on a frictionless, horizontal snow-covered plateau when she encounters a rough patch 3.65 m long. The coefficient of kinetic friction between this patch and her skis is 0.310. After crossing the rough patch and returning to friction-free snow, she skis down an icy, frictionless hill 3.50 m high.

Required:
a. How fast is the skier moving when she gets to the bottom of the hill?
b. How much internal energy was generated in crossing the rough patch?

Answers

Answer:

a) v = 3.71m/s

b) U = 616.71 J

Explanation:

a) To find the speed of the skier you take into account that, the work done by the friction surface on the skier is equal to the change in the kinetic energy:

[tex]-W_f=\Delta K=\frac{1}{2}m(v^2-v_o^2)\\\\-F_fd=\frac{1}{2}m(v^2-v_o^2)[/tex]  

(the minus sign is due to the work is against the motion of the skier)

m: mass of the skier = 58.0 kg

v: final speed = ?

vo: initial speed = 6.00 m/s

d: distance traveled by the skier in the rough patch = 3.65 m

Ff: friction force = Mgμ

g: gravitational acceleration = 9.8 m/s^2

μ: friction coefficient = 0.310

You solve the equation (1) for v:

[tex]v=\sqrt{\frac{2F_fd}{m}+v_o^2}=\sqrt{\frac{2mg\mu d}{m}+v_o^2}\\\\v=\sqrt{-2g\mu d+v_o^2}[/tex]

Next, you replace the values of all parameters:

[tex]v=\sqrt{-2(9.8m/s^2)(0.310)(3.65m)+(6.00m/s)^2}=3.71\frac{m}{s}[/tex]

The speed after the skier has crossed the roug path is 3.71m/s

b) The work done by the rough patch is the internal energy generated:

[tex]U=W_fd=F_fd=mg\mu d\\\\U=(58.0kg)(9.8m/s^2)(0.310)(3.50m)=616.71\ J[/tex]

The internal energy generated is 616.71J

A "swing" ride at a carnival consists of chairs that are swung in a circle by 19.8 m cables attached to a vertical rotating pole, as the drawing shows. Suppose the total mass of a chair and its occupant is 137 kg. (a) Determine the tension in the cable attached to the chair. (b) Find the speed of the chair.

Answers

Answer:

a)  T = 1342.6 cos θ, b)  v = 13.93 √(sin θ tan θ)

Explanation:

We can solve this problem using Newton's second law

Let's fix a reference system with a horizontal axis and the other vertical, therefore the only force to decompose is the tension, in these problems the most common is to measure the angle with respect to the vertical. Let's use trigonometry to find the components of the dispute

      cos θ = [tex]T_{y}[/tex] / T

      T_{y} = T cos tea

     sin θ = Tₓ / T

     Tₓ = T sin θ

let's write Newton's second law

axis and vertical

      T cos θ - W = 0

       T = mg / cos θ

let's calculate

      T = 137  9.8 cos θ

       T = 1342.6 cos θ

unfortunately there is no drawing or indication of the angle

Axis x Horizontal

       T sin θ = m a

acceleration is centripetal

        a = v² / R

        T sin θ = m v² / R

        v² = (g / cos θ) R sin θ

        v = √ (gR tan θ)

let's use trigonometry to find radius of gyration

          sin θ = R / L

          R = L sin θ

         v = √ (g L  sin θ tan θ)

let's calculate

         v = √(9.8 19.8 sin θ tant θ)

         v = 13.93 √(sin θ tan θ)

they do not give the angle for which the calculation cannot be finished

If the frequency is 5 Hz, determine the speed of the wave in the spring?? Can someone pls help me??

Answers

Answer:

    The speed of the wave is [tex]31.42 rad/s[/tex]

Explanation:

yes, we can.

Given data

frequency = 5 Hz

we know that the period T is expressed as

[tex]T= \frac{1}{f} \\[/tex]

Substituting we have

[tex]T= \frac{1}{5} \\T= 0.2s[/tex]

also the expression for angular velocity is

ω= [tex]\frac{2\pi}{T}[/tex]

Substituting we have

ω= [tex]\frac{2*3.142}{0.2}[/tex]

ω= [tex]\frac{6.284}{0.2} \\[/tex]

ω= [tex]31.42 rad/s[/tex]

Which scientist was the first to propose the heliocentric model of the universe

Answers

Answer:

Nicolaus Copernicus

Explanation:

Nevertheless, Copernicus began to work on astronomy on his own. Sometime between 1510 and 1514 he wrote an essay that has come to be known as the Commentariolus (MW 75–126) that introduced his new cosmological idea, the heliocentric universe, and he sent copies to various astronomers

To avoid breakdown of the capacitors, the maximum potential difference to which any of them can be individually charged is 125 V. Determine the maximum energy stored in the series combination.

Answers

Answer:

The maximum energy stored in the combination is 0.0466Joules

Explanation:

The question is incomplete. Here is the complete question.

Three capacitors C1-11.7 μF, C2 21.0 μF, and C3 = 28.8 μF are connected in series. To avoid breakdown of the capacitors, the maximum potential difference to which any of them can be individually charged is 125 V. Determine the maximum energy stored in the series combination.

Energy stored in a capacitor is expressed as E = 1/2CtV² where

Ct is the total effective capacitance

V is the supply voltage

Since the capacitors are connected in series.

1/Ct = 1/C1+1/C2+1/C3

Given C1 = 11.7 μF, C2 = 21.0 μF, and C3 = 28.8 μF

1/Ct = 1/11.7 + 1/21.0 + 1/28.8

1/Ct = 0.0855+0.0476+0.0347

1/Ct = 0.1678

Ct = 1/0.1678

Ct = 5.96μF

Ct = 5.96×10^-6F

Since V = 125V

E = 1/2(5.96×10^-6)(125)²

E = 0.0466Joules

It has been suggested that rotating cylinders about 12.5 mi long and 3.99 mi in diameter be placed in space and used as colonies. What angular speed must such a cylinder have so that the centripetal acceleration at its surface equals the free-fall acceleration on Earth

Answers

Answer:

The correct answer to the following question will be "0.0562 rad/s".

Explanation:

[tex]r =\frac{3.9}{2}\times 1609.34[/tex]

  [tex]=3138.213\ m[/tex]

As we know,

⇒  [tex]\omega^2 \ r=g[/tex]

On putting the values, we get

⇒  [tex]\omega^2\times 3138.213=9.8[/tex]

⇒  [tex]\omega = \sqrt{\frac{9.8}{3138.213}}[/tex]

⇒  [tex]\omega = 0.0562 \ rad /s[/tex]

High speed stroboscopic photographs show that the head of a 244 g golf club is traveling at 57.6 m/s just before it strikes a 45.2 g golf ball at rest on a tee. After the collision, the club head travels (in the same direction) at 39.9 m/s. Find the speed of the golf ball just after impact.

Answers

Answer:

The speed will be "1.06 m/s".

Explanation:

The given values are:

Momentum,

m1 = 244 g

m2 = 45.2 g

On applying momentum conservation ,

Let v2 become the final golf's speed.  

From Momentum Conservation

⇒  [tex]Total \ initial \ momentum = Total \ final \ momentum[/tex]

⇒  [tex]m1\times u1 + m2\times u2 = m1\times v1 + m2\times v2[/tex]

On putting the estimated values, we get

⇒  [tex]0.244\times 57.6+0=0.244\times 39.9+45.2\times v2[/tex]

⇒  [tex]57.844+0=9.7356+45.2\times v2[/tex]

⇒  [tex]48.1084=45.2\times v2[/tex]

⇒  [tex]v2=\frac{48.1084}{45.2}[/tex]

⇒  [tex]v2=1.06 \ m/s[/tex]

A 1 900-kg pile driver is used to drive a steel I-beam into the ground. The pile driver falls 4.00 m before coming into contact with the top of the beam, and it drives the beam 15.8 cm farther into the ground before coming to rest. Using energy considerations, calculate the average force the beam exerts on the pile driver while the pile driver is brought to rest.

Answers

Answer:

471392.4 N

Explanation:

From the question,

Just before contact with the beam,

mgh = Fd.................... Equation 1

Where m = mass of the beam, g = acceleration due to gravity, h = height. F =  average Force on the beam, d = distance.

make f the subject of the equation

F = mgh/d................ Equation 2

Given: m = 1900 kg, h = 4 m, d = 15.8 = 0.158 m

Constant: g = 9.8 m/s²

Substitute into equation 2

F = 1900(4)(9.8)/0.158

F = 471392.4 N

A 60-turn coil has a diameter of 13 cm. The coil is placed in a spatially uniform magnetic field of magnitude 0.60 T so that the face of the coil and the magnetic field are perpendicular. Find the magnitude of the emf induced in the coil (in V) if the magnetic field is reduced to zero uniformly in the following times.
(a) 0.80 S 0.5973V
(b) 8.0 s 5.973 Xv
(c) 70 S 6.838- 3 v

Answers

Answer:

a) 0.5985 V

b) 0.05985 V

c) 0.00684 V

Explanation:

Given that

Number of turn in the coil, N = 60 turns

Magnetic field of the coil, B = 0.6 T

Diameter of the coil, d = 0.13 m

If area is given as, πd²/4, then

A = π * 0.13² * 1/4

A = 0.0133 m²

The induced emf, ε = -N(dΦ*m) /dt

Note, Φm = BA

Substituting for Φ, we have

ε = -NBA/t.

Now, we substitute for numbers in the equation

ε = -(60 * 0.6 * 0.0133)/0.8

ε = 0.4788/0.8

ε = 0.5985 V

at 8s,

ε = -(60 * 0.6 * 0.0132)/8

ε = 0.4788/8

ε = 0.05985 V

at 70s

ε = -(60 * 0.6 * 0.0132)/70

ε = 0.4788/70

ε = 0.00684 V

Who first used the word atom to describe the smallest unit

Answers

Answer: It was Democritus, in fact, who first used the word atomos to describe the smallest possible particles of matter.

Explanation: hope this helped

At the equator, the earth's field is essentially horizontal; near the north pole, it is nearly vertical. In between, the angle varies. As you move farther north, the dip angle, the angle of the earth's field below horizontal, steadily increases. Green turtles seem to use this dip angle to determine their latitude. Suppose you are a researcher wanting to test this idea. You have gathered green turtle hatchlings from a beach where the magnetic field strength is 50 mu T and the dip angle is 56 degree. You then put the turtles in a 2.0 m diameter circular tank and monitor the direction in which they swim as you vary the magnetic field in the tank. You change the field by passing a current through a 50-tum horizontal coil wrapped around the tank. This creates a field that adds to that of the earth. In what direction should current pass through the coil, to produce a net field in the center of the tank that has a dip angle of 62 degree ? What current should you pass through the coil, to produce a net field in the center of the tank that has a dip angle of 62 degree ? Express your answer to two significant figures and include the appropriate units.

Answers

Answer:

Direction of current = clockwise

Magnitude of current, I = 0.36 A

Explanation:

The magnetic field strength, [tex]B_{E} = 50 \mu T[/tex]

The angle of dip, ∅ = 56°

The net magnetic field in the center of the tank is:

[tex]B_{net} = (B_{E} cos \phi ) (\hat{x} ) + ( B + B_{E} sin \phi)(-\hat{y})\\B_{net} = (50 cos 56 ) (\hat{x} ) + ( B +50 sin 56)(-\hat{y})\\B_{net} = (28 \mu T ) (\hat{x} ) + ( B +41.4 \mu T)(-\hat{y})\\[/tex]

The direction of the net magnetic field is:

[tex]\phi = tan^{-1} \frac{B + 41.4 }{28} \\tan \phi = \frac{B + 41.4 }{28}\\\phi = 62^0\\tan 62 = \frac{B + 41.4 }{28}\\28 tan 62 = B + 41.4\\52.66 = B + 41.4\\B = 11.26 \mu T[/tex]

The magnetic field due to the coil:

[tex]B = \frac{\mu_{0}NI }{2r} \\11.26 * 10^{-6} = \frac{4\pi * 10^{-7} * 50 *I }{2 *1}\\I = \frac{2 * 11.26 * 10^{-6}}{4\pi * 10^{-7} * 50} \\I = 0.36 A[/tex]

The current must be in clockwise direction to produce the field in downward direction

A carnot heat engine has an efficiency of 0.800. if it operates between a deep lake with a constant temperature of 280.0 k and a hot reservoir, what is the temperature of the hot reservoir?

Answers

i believe that the answer is

Two small plastic spheres each have a mass of 2.0 g and a charge of −50.0 nC. They are placed 2.0 cm apart (center to center). What is the magnitude of the electric force on each sphere? By what factor is the electric force on a sphere larger than its weight?

Answers

Answer:

a) F = 0.0561 N

b) F = 2.86*W

Explanation:

a) The magnitude of the electric force between the plastic spheres is given by the following formula:

[tex]F=k\frac{q_1q_2}{r^2}[/tex]    (1)

k: Coulomb's constant = 8.98*10^9 Nm^2/C^2

q1 = q2: charge of the plastic spheres = -50.0nC = -50.0*10^-9 C

r: distance between the plastic spheres = 2.0 cm = 0.02 m

You replace the values of the parameters in the equation (1):

[tex]F=(8.98*10^9Nm^2/C^2)\frac{(-50.0*10^{-9}C)^2}{(0.02m)^2}\\\\F=0.0561N[/tex]

The electric force between the spheres is 0.0561 N

b) To calculate the relation between weight and electric force, you first calculate the weight of one of the spheres:

[tex]W=mg[/tex]

m: mass = 2.0g = 2.0*10^-3 kg

g: gravitational acceleration = 9.8 m/s^2

[tex]W=(2.0*10^{-3}kg)(9.8m/s^2)=0.0196N[/tex]

The ratio between W and F is:

[tex]\frac{F}{W}=\frac{0.0561N}{0.0196N}=2.86\\\\F=2.86W[/tex]

The electric force is 2.86 times the weight

(a) The magnitude of the electric force on each sphere is [tex]5.625 \times 10^{-2} \ N[/tex].

(b)  The electric force on a sphere is larger than its weight by 2.87.

The given parameters:

mass of each sphere, m = 2.0 gcharge on each sphere, q = -50 nCdistance between the charges, d = 2.0 cm

The magnitude of the electric force on each sphere is calculated as follows;

[tex]F = \frac{kq^2}{r^2} \\\\F = \frac{9\times 10^9 \times (5 0 \times 10^{-9})^2}{(0.02)^2} \\\\F = 5.625 \times 10^{-2} \ N[/tex]

The weight of a sphere is calculated as follows;

[tex]W = mg\\\\W = 0.002 \times 9.8\\\\W = 0.0196 \ N[/tex]

Compare the electric force and the weight of a sphere;

[tex]= \frac{F}{W} = \frac{5.625 \times 10^{-2}}{0.0196} = 2.87[/tex]

Thus, the electric force on a sphere is larger than its weight by 2.87.

Learn more here:https://brainly.com/question/12533085

A thin plastic rod of length 2.6 m is rubbed all over with wool, and acquires a charge of 98 nC, distributed uniformly over its surface. Calculate the magnitude of the electric field due to the rod at a location 13 cm from the midpoint of the rod. Do the calculation two ways, first using the exact formula for a rod of any length, and second using the approximate formula for a long rod.

Answers

Answer:

By exact formula

5076.59N/C

And by approximation formula

5218.93N/C

Explanation:

We are given that

Length of rod,L=2.6 m

Charge,q=98nC=[tex]98\times 10^{-9} C[/tex]

[tex]1nC=10^{-9} C[/tex]

a=13 cm=0.13 m

1 m=100 cm

By exact formula

The magnitude of  the electric field due to the rod at a location 13 cm from the midpoint of the rod=[tex]\frac{kq}{a}\times \frac{1}{\sqrt{a^2+\frac{L^2}{4}}}[/tex]

Where k=[tex]9\times 10^9[/tex]

Using the formula

The magnitude of  the electric field due to the rod at a location 13 cm from the midpoint of the rod=[tex]\frac{9\times 10^9\times 98\times 10^{-9}}{0.13}\times \frac{1}{\sqrt{(0.13)^2+\frac{(2.6)^2}{4}}}=5076.59N/C[/tex]

In approximation formula

a<<L

[tex]a^2+(\frac{L}{2})^2=\frac{L^2}{4}[/tex]

Therefore,the magnitude of  the electric field due to the rod at a location 13 cm from the midpoint of the rod=[tex]\frac{kq}{a}\times \frac{1}{\sqrt{\frac{L^2}{4}}}[/tex]

The magnitude of  the electric field due to the rod at a location 13 cm from the midpoint of the rod=[tex]\frac{9\times 10^9\times 98\times 10^{-9}}{0.13}\times \frac{1}{\sqrt{\frac{(2.6)^2}{4}}}=5218.93N/C[/tex]

Other Questions
A car 4m long moving at a velocity of 25m/s was beside a lorry 20m long with a velocity 19mls. At t = 0, the distance between them was 10m. How long will it take the car to overtake the lorry (a) 9s (b) 5s (c) 3s (d) 2s operates department stores in numerous states. Suppose selected financial statement data (in millions) for 2020 are presented below. End of Year Beginning of Year Cash and cash equivalents $ 770 $ 69 Accounts receivable (net) 1,950 1,880 Inventory 810 860 Other current assets 590 331 Total current assets $4,120 $3,140 Total current liabilities $2,030 $1,640 For the year, net credit sales were $8,258 million, cost of goods sold was $5,328 million, and net cash provided by operating activities was $1,251 million. Compute the current ratio, accounts receivable turnover, average collection period, inventory turnover and days in inventory at the end of the current year. (Round current ratio to 2 decimal places, e.g. 1.83 and all other answers to 1 decimal place, e.g. 1.8. Use 365 days for calculation.) Current ratio :1 Accounts receivable turnover times Average collection period days Inventory turnover times Days in inventory days Antonio's checking account currently has a negative balance of $42.17. Yesterday he made purchases of $3.75 and $12.07. If Antonio's bank charges $34 for overdraft protection, what was his account balance prior to yesterday's transactions? again i don't understand :( which graph function of x? You spin the spinner and flip a coin. Find the probability of the compound event.The probability of spinning an even number and flipping heads is Suppose Eric and Isaac both drive the same car, and have the samedeductible for car insurance. If Eric is five years younger, who is most likely topay a higher annual premium?OA. IsaacOB. Neither, they pay the same.OC. Can't tell from the information givenOD. Eric NO ONE IS ANSWERING ME:( BRAINLIEST ANSWER GIVEN A frame shop charges according to both of framing needed to surround the picture and the amount of glass needed to cover the picture. A. Find the area and perimeter of the trapezoid-shaped framed picture to the right. B. Identify whether the frame has to do with perimeter or area and the same with the glass. lengths are 30 ft each, widths are 27 & 63, height is 24 Whats the correct answer for this question? mary thinks the triangle is equilateral. how would you support or dispute her conjecture? Please answer this correctly if x = 5, what is ((x+3)/4)*234 What is the potential energy of two charges of +4.6 C and +1.0 C that are separated by a distance of 10.0 cm? A student uses the right-hand rule as shown.What is the direction of the magnetic field in front of the wireclosest to the student?uprightdownleft PLEASE HELP ASAP!!!Read the incomplete sentence and choose the option with the correct verbs to complete it.La seora Vila ________ tan desagradable! Sus sobrinos no _______ en su casa para ayudar a la seora Vila.A. es; estnB. estamos; somosC. somos; estnD.soy; estoy Nickie has been experiencing frequent periods of anxiety and occasional attacks of rapid breathing, increased heart rate, sweating, and trembling. Her doctor diagnosed her with ______. Mr Smith had four daughters.Each daughter has four brothers.How many children does Mr Smith have During May 2020, a company spend $14,000 on ordinary maintenance of its delivery trucks. These maintenance costs should have been recorded as debits to Maintenance Expense but the company mistakenly capitalized these costs instead. Which of the following is true?A. Total Assets are overstated as of May 31, 2020 and May 2020 Net Income is overstated. B. Total Assets are overstated as of May 31, 2020 and May 2020 Net Income is properly stated. C. Total Assets are understated as of May 31, 2020 and May 2020 Net Income is understated. D. Both Total Assets as of May 31, 2020 and May 2020 Net Income are properly stated. 16b2c120.25 Factor Completey A box contains five keys, only one of which will open a lock. Keys are randomly selected and tried, one at a time, until the lock is opened (keys that do not work are discarded before another is tried). Let Y be the number of the trial on which the lock is opened. Find the probability function for Y.